LSAT and Law School Admissions Forum

Get expert LSAT preparation and law school admissions advice from PowerScore Test Preparation.

User avatar
 Dave Killoran
PowerScore Staff
  • PowerScore Staff
  • Posts: 5850
  • Joined: Mar 25, 2011
|
#84918
Complete Question Explanation
(The complete setup for this game can be found here: lsat/viewtopic.php?t=6605)

The correct answer choice is (B)

The question stem establishes that Q and S both run in the meet, and from the first rule we can infer that T must also run in the meet, immediately after Q. Because the second rule limits S to running in just the first or third races, let’s examine what could occur in each scenario:

               If S runs in the first race, then R could run in the second race, and Q and T could run in the
               third and fourth races (S-R-Q-T). While this is just one solution with S running in the first
               race, it proves that S can run first.

               If S runs in the third race, then the QT block must run in the first two races. U then runs in
               the fourth race (Q-T-S-U). This proves that Q can run in the first race.

As we have explored all possibilities for who can be first based on the possible positions of S, we can determine that only Q and S can run first, and thus answer choice (B) is correct.

Get the most out of your LSAT Prep Plus subscription.

Analyze and track your performance with our Testing and Analytics Package.